LSAT and Law School Admissions Forum

Get expert LSAT preparation and law school admissions advice from PowerScore Test Preparation.

 Administrator
PowerScore Staff
  • PowerScore Staff
  • Posts: 8916
  • Joined: Feb 02, 2011
|
#81044
Complete Question Explanation

Justify the Conclusion, Principle. The correct answer choice is (A).

Answer choice (A): This is the correct answer choice.

Answer choice (B):

Answer choice (C):

Answer choice (D):

Answer choice (E):

This explanation is still in progress. Please post any questions below!
 mokkyukkyu
  • Posts: 97
  • Joined: Aug 17, 2016
|
#28675
Hi,

So I was just wondering...what we need to know is only "Shimada does not criticize his tardiness" right?
There are other information in A, and those are not necessary for justifying the application right?
 Adam Tyson
PowerScore Staff
  • PowerScore Staff
  • Posts: 5153
  • Joined: Apr 14, 2011
|
#29210
Right! This is conditional - if Criticize Self or Vow to Stop, then Criticize Others. In the application, we have met one of the sufficient conditions (not vowing to stop) and have claimed that the necessary condition is true (Shimada should not criticize McFeney). In order to justify that conclusion we have to have the other sufficient condition in place - Shimada has to fail to criticize himself. Answer A supplies that and thus justifies the conclusion.
 pavalos5777
  • Posts: 10
  • Joined: Jun 17, 2017
|
#39388
Hello, I eliminated answer choice A because the sufficient condition contains an "OR"--I took this to mean that as long as one condition is met then the necessary condition stands. That is, Shimada did not vow to stop, so whether he criticizes himself or not, he should not criticize McFeney.

I then went with E, my second contender, since it demonstrates that their behavior is comparable.

Can you please clarify this for me?

Thank you,
Pierre
 Eric Ockert
PowerScore Staff
  • PowerScore Staff
  • Posts: 164
  • Joined: Sep 28, 2011
|
#39567
Hi Pierre!

I think you are right on that. It seems that you have met one of the two options on the sufficient condition (not vowing to stop a form of behavior in oneself), and therefore the necessary (he should not criticize that form of behavior in another) would follow. However, what is in doubt here is whether the "tardiness" is actually a "form of behavior in oneself."

Notice...what if Shimada was never actually tardy? Any vow to stop being tardy would not really be a vow to stop a "form of behavior in oneself." That phrase implies that we must be talking about something someone actually engages in. But with answer choice (A), if both of these two are "regularly tardy", now we have a "form of behavior" which they both possess. So, the fact that Shimada does not vow to stop being tardy (which he now regularly is) would prove that he should not criticize McFeney (who also is regularly tardy).

The additional information, that he criticizes this behavior "without criticizing his own", is really just extra. You probably have justification without it, but it certainly doesn't hurt anything about answer (A). Remember, with Justify the Conclusion questions, your correct answer can give you more than enough to prove, it just can't give you less. And in this case, that extra info is probably there to make you feel as though you are being tricked.

Definitely a sneaky question though.

Hope that helps!
 pavalos5777
  • Posts: 10
  • Joined: Jun 17, 2017
|
#39621
Thanks Eric, it definitely succeeding in making me feel tricked. Your response was helpful, it will add to my filter for this question type.

Best,
Pierre
 reg4315
  • Posts: 12
  • Joined: Sep 13, 2017
|
#41980
Hi Everyone!

Just want some clarification on this one -

I selected A, but came at it with the same understanding as Pavalos did (above). Isn't the application already justified since one of the two sufficient conditions have been met? Given that it's an "or" statement, do we need to meet both for the application of the principle to be fully justified? I'm asking more generally for the future, because in my head, the application was already entirely justified, since one of the two sufficient conditions in the "OR" clause was met, triggering the fulfillment of the necessary, as confirmed in the stimulus. Are there different degrees as to how "much" this can be justified? Is it more justified because with ac A, we meet both suff. conditions? Is it any less justified if we only meet one of the two?


Than you!
 Francis O'Rourke
PowerScore Staff
  • PowerScore Staff
  • Posts: 471
  • Joined: Mar 10, 2017
|
#42098
Hi Reg,

The application is written in a rather subtle and tricky way. Although the application seems to accuse Shimada of tardiness, on closer inspection we do not have any proof of that. We are only told that Shimada has not vowed to stop being tardy himself. It may still be the case that Shimada is always on time, which means that this behavior is not one that he engages in.
 harvoolio
  • Posts: 63
  • Joined: Apr 25, 2018
|
#45957
So is the stimulus like a loaded question in assuming that the action is taking place without establishing so i.e. the loaded question of "Have you stopped beating your wife?" assumes that you beat your wife in the first place?

Thanks.
 Adam Tyson
PowerScore Staff
  • PowerScore Staff
  • Posts: 5153
  • Joined: Apr 14, 2011
|
#46962
That's an interesting way of looking at it, harvoolio! I'll say yes, it is like that - the application of the principle is valid if Shimada is sometimes tardy and does not criticize himself for it, so the author appears to have assumed that to be the case. Adding that info would justify the conclusion.

The difference between your example and this question is that in your example, there is no principle or application thereof, just a question that assumes a precondition to be true. But it is similar in that it makes a certain assumption which, if true, would justify asking the question.

Get the most out of your LSAT Prep Plus subscription.

Analyze and track your performance with our Testing and Analytics Package.